data icon indicating copy to clipboard operation
data copied to clipboard

2-Markov Menger and cocountable subsets of reals

Open Moniker1998 opened this issue 6 months ago • 3 comments

@StevenClontz @ccaruvana

I read some of Steven's article https://dml.cz/bitstream/handle/10338.dmlcz/146790/CommentatMathUnivCarolRetro_58-2017-2_8.pdf where he proves that a certain set-theoretic assumption implies 2-Markov Menger for Fortissimo space on $\mathbb{R}$. Same assumption works for cocountable topology on $\mathbb{R}$.

So I've asked if this space having 2-Markov Menger property is independent of ZFC:

https://mathoverflow.net/questions/492388/does-cocountable-topology-on-mathbbr-have-the-2-markov-menger-property

And if the assumption is independent of ZFC:

https://math.stackexchange.com/questions/5063919/is-mathcala-mathfrakc-independent-of-mathrmzfc

All of the properties of spaces up to S30 except for S17, S18, S22 having 2-Markov Menger property, are either done or equivalent to continuum hypothesis (as checked by me).

I realize this might be a hard problem, but I wanted to get your attention on this one. Perhaps some collaboration to solve this problem will be possible in the future, at least.

Moniker1998 avatar May 26 '25 17:05 Moniker1998

I've got a comment on my question, and it looks like @StevenClontz and Dow show in a paper from 2018 that there is a model of ZFC with $\mathfrak{c} = \aleph_2$ and $\lnot\mathcal{A}(\aleph_2)$, and so $\mathcal{A}(\mathfrak{c})$ is independent of ZFC.

I suppose it'd be useful to have gone through more than just one paper. 😅

Moniker1998 avatar Jun 03 '25 14:06 Moniker1998

Yeah, sorry I didn't have a chance to carefully respond and point you to this paper (in which all the set theory was due to Dow).

StevenClontz avatar Jun 06 '25 15:06 StevenClontz

@StevenClontz that's okay. Some high school student used AI and pointed me to that paper (I normally don't use AI but I guess its useful for that). If you want to write a short answer, doesn't have to be careful at all, to that https://math.stackexchange.com/questions/5063919/is-mathcala-mathfrakc-independent-of-mathrmzfc question I made, feel free to - it would be nice to have an answer from the authority itself :) Otherwise it'd feel a bit wasteful of the bounty I have on it

Moniker1998 avatar Jun 06 '25 15:06 Moniker1998